परिमित संभावित स्रोत: Difference between revisions

From Vigyanwiki
(Created page with "परिमित संभावित कुँआ (परिमित वर्ग कुँआ के रूप में भी जाना जाता है) क...")
 
No edit summary
 
(13 intermediate revisions by 4 users not shown)
Line 1: Line 1:
परिमित संभावित कुँआ (परिमित वर्ग कुँआ के रूप में भी जाना जाता है) [[क्वांटम यांत्रिकी]] की एक अवधारणा है। यह अनंत क्षमता वाले कुएं का विस्तार है, जिसमें एक कण एक बॉक्स तक ही सीमित है, लेकिन जिसकी संभावित [[ऊर्जा]] दीवारें सीमित हैं। अनंत क्षमता वाले कुएं के विपरीत, कण के बॉक्स के बाहर पाए जाने से जुड़ी एक [[संभावना]] है। क्वांटम यांत्रिक व्याख्या शास्त्रीय व्याख्या के विपरीत है, जहां यदि कण की कुल ऊर्जा दीवारों की [[संभावित ऊर्जा]] बाधा से कम है तो इसे बॉक्स के बाहर नहीं पाया जा सकता है। क्वांटम व्याख्या में, कण के बॉक्स के बाहर होने की गैर-शून्य संभावना होती है, भले ही कण की ऊर्जा दीवारों की संभावित ऊर्जा बाधा (सीएफ [[क्वांटम टनलिंग]]) से कम हो।
'''परिमित संभावित स्रोत''' ('''परिमित वर्ग स्रोत''' के रूप में भी जाना जाता है) [[क्वांटम यांत्रिकी]] की अवधारणा होती है। यह अनंत क्षमता वाले कुएं का विस्तार होता है, जिसमें कण '''"बॉक्स"''' तक ही सीमित होता है, किन्तु जिसकी संभावित [[ऊर्जा]] '''"दीवारें"''' सीमित होती हैं। इस प्रकार अनंत क्षमता वाले कुएं के विपरीत, कण के बॉक्स के बाहर पाए जाने से जुड़ी [[संभावना]] होती है। चूँकि क्वांटम यांत्रिक व्याख्या मौलिक व्याख्या के विपरीत होती है, जहां यदि कण की कुल ऊर्जा दीवारों की [[संभावित ऊर्जा]] बाधा से कम है तब इसे बॉक्स के बाहर नहीं पाया जा सकता है। इस प्रकार क्वांटम व्याख्या में, कण की ऊर्जा दीवारों की संभावित ऊर्जा बाधा (सीएफ [[क्वांटम टनलिंग]]) से कम होने पर भी कण के बॉक्स के बाहर होने की गैर-शून्य संभावना होती है।


==एक-आयामी बॉक्स में कण==
=='''एक-आयामी बॉक्स में कण'''==
एक्स-अक्ष पर 1-आयामी मामले के लिए, समय-स्वतंत्र श्रोडिंगर समीकरण को इस प्रकार लिखा जा सकता है:
एक्स-अक्ष पर 1-आयामी स्थितियों के लिए, समय-स्वतंत्र श्रोडिंगर समीकरण को इस प्रकार लिखा जा सकता है।
{{NumBlk||<math display="block"> -\frac{\hbar^2}{2 m} \frac{d^2 \psi}{d x^2} + V(x) \psi = E \psi </math>|{{EquationRef|1}}}}
{{NumBlk||<math display="block"> -\frac{\hbar^2}{2 m} \frac{d^2 \psi}{d x^2} + V(x) \psi = E \psi </math>|{{EquationRef|1}}}}
कहाँ
जहाँ
*<math>\hbar = \frac{h}{2 \pi}</math> घटा हुआ प्लैंक स्थिरांक है,
*<math>\hbar = \frac{h}{2 \pi}</math> घटा हुआ प्लैंक स्थिरांक होता है,
*<math>h </math> प्लैंक स्थिरांक है,
*<math>h </math> प्लैंक स्थिरांक होता है,
*<math>m </math> कण का [[द्रव्यमान]] है,
*<math>m </math> कण का [[द्रव्यमान]] होता है,
*<math>\psi</math> वह (जटिल मूल्यवान) तरंग [[तरंग क्रिया]] है जिसे हम खोजना चाहते हैं,
*<math>\psi</math> वह (समष्टि मूल्यवान) [[तरंग क्रिया]] होती है जिसे हम खोजना चाहते हैं,
*<math>V(x)</math> प्रत्येक बिंदु x पर संभावित ऊर्जा का वर्णन करने वाला एक फ़ंक्शन है, और
*<math>V(x)</math> प्रत्येक बिंदु एक्स पर संभावित ऊर्जा का वर्णन करने वाला फलन होता है, और
*<math>E</math> ऊर्जा है, एक वास्तविक संख्या, जिसे कभी-कभी आइजेनएनर्जी भी कहा जाता है।
*<math>E</math> ऊर्जा होती है, वास्तविक संख्या, जिसे कभी-कभी आइजेनएनर्जी भी कहा जाता है।


लंबाई L के 1-आयामी बॉक्स में कण के मामले में, क्षमता है <math>V_0</math> बॉक्स के बाहर, और बीच में x के लिए शून्य <math>-L/2</math> और <math>L/2</math>. वेवफ़ंक्शन को x की विभिन्न श्रेणियों पर अलग-अलग वेवफ़ंक्शन से बना माना जाता है, यह इस पर निर्भर करता है कि x बॉक्स के अंदर है या बाहर। इसलिए, वेवफ़ंक्शन को इस प्रकार परिभाषित किया गया है:
लंबाई एल के 1-आयामी बॉक्स में कण की स्थितियों में, <math>V_0</math> क्षमता होती है। इस प्रकार बॉक्स के बाहर, और मध्य में एक्स के लिए शून्य <math>-L/2</math> और <math>L/2</math>. तरंग फलन को एक्स की विभिन्न श्रेणियों पर भिन्न-भिन्न तरंग फलन से बना माना जाता है, यह इस पर निर्भर करता है कि एक्स बॉक्स के अंदर या बाहर होता है। इसलिए, तरंग फलन को इस प्रकार परिभाषित किया गया है।


<math display="block">\psi = \begin{cases}
<math display="block">\psi = \begin{cases}
Line 18: Line 18:
\psi_2, & \text{if }-L/2<x<L/2\text{ (the region inside the box)} \\
\psi_2, & \text{if }-L/2<x<L/2\text{ (the region inside the box)} \\
\psi_3, & \text{if }x>L/2\text{  (the region outside the box)}
\psi_3, & \text{if }x>L/2\text{  (the region outside the box)}
\end{cases}</math>
\end{cases}</math>बॉक्स के अंदर के क्षेत्र के लिए, वी(एक्स) = 0 और समीकरण 1 कम हो जाता है<math display="block">-\frac{\hbar^2}{2 m} \frac{d^2 \psi_2}{d x^2} = E \psi_2 .</math>
दे<math display="block">k = \frac{\sqrt{2mE}}{\hbar},</math>समीकरण बन जाता है<math display="block">\frac{d^2 \psi_2}{d x^2} = -k^2 \psi_2 .</math>यह सामान्य समाधान के साथ अच्छी प्रकार से अध्ययन किया गया [[अंतर समीकरण]] और [[eigenvectors|आइजेनवेक्टर]] समस्या होती है<math display="block">\psi_2 = A \sin(kx) + B \cos(kx)\, .</math>
इस प्रकार,<math display="block">E = \frac{k^2 \hbar^2}{2m} .</math>




===बॉक्स के अंदर===
यहां, और बी कोई भी सम्मिश्र संख्या हो सकती हैं, और "के" कोई भी वास्तविक संख्या हो सकती है।
बॉक्स के अंदर के क्षेत्र के लिए, V(x) = 0 और समीकरण 1 कम हो जाता है
<math display="block">-\frac{\hbar^2}{2 m} \frac{d^2 \psi_2}{d x^2} = E \psi_2 .</math>
दे
<math display="block">k = \frac{\sqrt{2mE}}{\hbar},</math>
समीकरण बन जाता है
<math display="block">\frac{d^2 \psi_2}{d x^2} = -k^2 \psi_2 .</math>
यह एक सामान्य समाधान के साथ एक अच्छी तरह से अध्ययन किया गया [[अंतर समीकरण]] और [[eigenvectors]] समस्या है
<math display="block">\psi_2 = A \sin(kx) + B \cos(kx)\, .</math>
इस तरह,
<math display="block">E = \frac{k^2 \hbar^2}{2m} .</math>
यहां, A और B कोई भी सम्मिश्र संख्या हो सकते हैं, और k कोई भी वास्तविक संख्या हो सकती है।


=== बॉक्स के बाहर ===
=== बॉक्स के बाहर ===
बॉक्स के बाहर के क्षेत्र के लिए, चूँकि क्षमता स्थिर है, <math>V(x) = V_0</math> और समीकरण {{EquationNote|1}} बन जाता है:
बॉक्स के बाहर के क्षेत्र के लिए <math>V(x) = V_0</math> और समीकरण {{EquationNote|1}} बन जाता है, चूँकि क्षमता स्थिर होती है।
<math display="block">-\frac{\hbar^2}{2 m} \frac{d^2 \psi_1}{d x^2} = ( E - V_0) \psi_1 </math>
<math display="block">-\frac{\hbar^2}{2 m} \frac{d^2 \psi_1}{d x^2} = ( E - V_0) \psi_1 </math>
समाधान के दो संभावित परिवार हैं, यह इस पर निर्भर करता है कि E इससे कम है या नहीं <math>V_0</math> (कण विभव में बंधा हुआ है) अथवा E से अधिक है <math>V_0</math> (कण स्वतंत्र है).
सामान्यतः समाधान के दो संभावित समूह होते हैं, यह इस पर निर्भर करता है कि इससे कम होता है या नहीं होता है <math>V_0</math> (कण विभव में बंधा हुआ है) अथवा से अधिक <math>V_0</math> (कण स्वतंत्र) होता है।


एक मुक्त कण के लिए, <math>E > V_0</math>, और देना <math display="block">k' = \frac{\sqrt{2m(E - V_0)}}{\hbar}</math> का उत्पादन
मुक्त कण के लिए, <math>E > V_0</math>, और देना <math display="block">k' = \frac{\sqrt{2m(E - V_0)}}{\hbar}</math> का उत्पादन
<math display="block">\frac{d^2 \psi_1}{d x^2} = -k'^2 \psi_1 </math>
<math display="block">\frac{d^2 \psi_1}{d x^2} = -k'^2 \psi_1 </math>
इनसाइड-वेल केस के समान समाधान फॉर्म के साथ:
आंतरिक अच्छी प्रकार की स्थिति के समान समाधान फॉर्म के साथ:


<math display="block">\psi_1 = C \sin(k' x) + D \cos(k' x) </math>
<math display="block">\psi_1 = C \sin(k' x) + D \cos(k' x) </math>
यह विश्लेषण बाध्य स्थिति पर ध्यान केंद्रित करेगा, जहां <math>E < V_0</math>. दे
यह विश्लेषण बाध्य स्थिति पर ध्यान केंद्रित करता है, जहां <math>E < V_0</math> देता है।
<math display="block">\alpha = \frac{\sqrt{2m(V_0 - E)}}{\hbar}</math>
<math display="block">\alpha = \frac{\sqrt{2m(V_0 - E)}}{\hbar}</math>
का उत्पादन
का उत्पादन
<math display="block">\frac{d^2 \psi_1}{d x^2} = \alpha^2 \psi_1 </math>
<math display="block">\frac{d^2 \psi_1}{d x^2} = \alpha^2 \psi_1 </math>
जहां सामान्य समाधान घातीय है:
जहां सामान्य समाधान घातीय होता है।
<math display="block">\psi_1 = Fe^{- \alpha x}+ Ge^{ \alpha x} </math>
<math display="block">\psi_1 = Fe^{- \alpha x}+ Ge^{ \alpha x} </math>
इसी प्रकार, बॉक्स के बाहर दूसरे क्षेत्र के लिए:
इसी प्रकार, बॉक्स के बाहर दूसरे क्षेत्र के लिए:


<math display="block">\psi_3 = He^{- \alpha x}+ Ie^{ \alpha x} </math>
<math display="block">\psi_3 = He^{- \alpha x}+ Ie^{ \alpha x} </math>
अब मौजूदा समस्या का विशिष्ट समाधान खोजने के लिए, हमें उपयुक्त सीमा शर्तों को निर्दिष्ट करना होगा और ए, बी, एफ, जी, एच और आई के लिए मान ढूंढना होगा जो उन शर्तों को पूरा करते हैं।
वर्तमान उपस्तिथ समस्या का विशिष्ट समाधान खोजने के लिए, हमें उपयुक्त सीमा शर्तों को निर्दिष्ट करना होता है और ए, बी, एफ, जी, एच और आई के लिए मान खोजना होता है, जो उन शर्तों को पूर्ण करते हैं।


===बाउंड अवस्था के लिए वेवफंक्शन ढूँढना===
===बाउंड अवस्था के लिए तरंग फलन खोजना===
श्रोडिंगर समीकरण के समाधान निरंतर और निरंतर भिन्न होने चाहिए।<ref>{{harvnb|Hall|2013}} Proposition 5.1</ref> ये आवश्यकताएं पहले से प्राप्त अंतर समीकरणों पर सीमा की स्थिति हैं, यानी, कुएं के अंदर और बाहर के समाधानों के बीच मिलान की स्थिति।
श्रोडिंगर समीकरण के समाधान निरंतर और निरंतर भिन्न होते है।<ref>{{harvnb|Hall|2013}} Proposition 5.1</ref> यह आवश्यकताएं पहले से प्राप्त अंतर समीकरणों पर सीमा की स्थिति होती हैं, अर्थात् कुएं के अंदर और बाहर के समाधानों के मध्य मिलान की स्थिति होती है।


इस मामले में, परिमित संभावित कुआं सममित है, इसलिए आवश्यक गणनाओं को कम करने के लिए समरूपता का उपयोग किया जा सकता है।
इस स्थितियों में, परिमित संभावित कुआं सममित होता है, इसलिए आवश्यक गणनाओं को कम करने के लिए समरूपता का उपयोग किया जा सकता है।


पिछले अनुभागों का सारांश:
पिछले अनुभागों का सारांश:
Line 65: Line 56:
\psi_2, & \text{if }-L/2< x< L/2\text{ (the region inside the box)} \\
\psi_2, & \text{if }-L/2< x< L/2\text{ (the region inside the box)} \\
\psi_3 & \text{if }x>L/2\text{  (the region outside the box)}
\psi_3 & \text{if }x>L/2\text{  (the region outside the box)}
\end{cases}</math>
\end{cases}</math>जहां हमें <math>\psi_1</math>, <math>\psi_2 </math>, और <math>\psi_3 </math> प्राप्त होता है।
जहां हमने पाया <math>\psi_1</math>, <math>\psi_2 </math>, और <math>\psi_3 </math> होना:
 
<math display="block">\begin{align}
<math display="block">\begin{align}
\psi_1 &= Fe^{- \alpha x}+ Ge^{ \alpha x} \\
\psi_1 &= Fe^{- \alpha x}+ Ge^{ \alpha x} \\
Line 72: Line 63:
\psi_3 &= He^{- \alpha x}+ Ie^{ \alpha x}
\psi_3 &= He^{- \alpha x}+ Ie^{ \alpha x}
\end{align}</math>
\end{align}</math>
हम इसे ऐसे देखते हैं <math>x</math> जाता है <math>-\infty</math>, द <math>F</math> पद अनंत तक जाता है. इसी तरह, जैसे <math>x</math> जाता है <math>+\infty</math>, द <math>I</math> पद अनंत तक जाता है. तरंग फलन को वर्गाकार समाकलनीय बनाने के लिए, हमें सेट करना होगा <math>F = I = 0</math>, और हमारे पास है:
<math display="block">\psi_1 = Ge^{ \alpha x} </math> और <math display="block">\psi_3 = He^{- \alpha x} </math>
अगला, हम जानते हैं कि समग्र <math>\psi </math> फ़ंक्शन निरंतर और भिन्न होना चाहिए। दूसरे शब्दों में, फ़ंक्शंस और उनके डेरिवेटिव के मान विभाजन बिंदुओं पर मेल खाने चाहिए:


{| cellpadding=4
हम इसे ऐसे देखते हैं. जंहा <math>x</math> जाता है <math>-\infty</math> तक, जंहा <math>F</math> पद अनंत तक जाता है. इसी प्रकार, जैसे <math>x</math> जाता है <math>+\infty</math> तक, उसी प्रकार <math>I</math> पद अनंत तक जाता है। सामान्यतः तरंग फलन को वर्गाकार समाकलनीय बनाने के लिए, हमें समुच्चय <math>F = I = 0</math> करना होता है, और हमारे पास होता है।
| <math>\psi_1(-L/2) = \psi_2(-L/2) </math> || || <math>\psi_2(L/2) = \psi_3(L/2) </math>
<math display="block">\psi_1 = Ge^{ \alpha x} </math> और<math display="block">\psi_3 = He^{- \alpha x} </math>
 
 
अगला, हम जानते हैं कि समग्र <math>\psi </math> फलन निरंतर और भिन्न होता है। दूसरे शब्दों में, फलन और उनके व्युत्पन्न के मान विभाजन बिंदुओं पर मेल खाते है।
{| cellpadding="4"
| <math>\psi_1(-L/2) = \psi_2(-L/2) </math> || || <math>\psi_2(L/2) = \psi_3(L/2) </math>
|-  
|-  
| <math>\left.\frac{d\psi_1}{dx}\right|_{x=-L/2} = \left.\frac{d\psi_2}{dx}\right|_{x=-L/2} </math> || || <math>\left.\frac{d\psi_2}{dx}\right|_{x=L/2} = \left.\frac{d\psi_3}{dx}\right|_{x=L/2} </math>
| <math>\left.\frac{d\psi_1}{dx}\right|_{x=-L/2} = \left.\frac{d\psi_2}{dx}\right|_{x=-L/2} </math> || || <math>\left.\frac{d\psi_2}{dx}\right|_{x=L/2} = \left.\frac{d\psi_3}{dx}\right|_{x=L/2} </math>
|}
|}
इन समीकरणों के दो प्रकार के समाधान हैं, सममित, जिसके लिए <math>A = 0</math> और <math>G = H</math>, और एंटीसिमेट्रिक, जिसके लिए <math>B = 0</math> और <math>G=-H</math>. सममित मामले के लिए हमें मिलता है
इन समीकरणों के दो प्रकार के समाधान होते हैं, अतः सममित, जिसके लिए <math>A = 0</math> और <math>G = H</math>, और एंटीसिमेट्रिक, जिसके लिए <math>B = 0</math> और <math>G=-H</math>. सममित स्थितियों के लिए हमें मिलता है।


<math display="block"> He^{- \alpha L/2} = B \cos(k L/2)</math>
<math display="block"> He^{- \alpha L/2} = B \cos(k L/2)</math><math display="block"> - \alpha He^{- \alpha L/2} = - k B \sin(k L/2)</math>
<math display="block"> - \alpha He^{- \alpha L/2} = - k B \sin(k L/2)</math>
तब अनुपात लेने से मिलता है
तो अनुपात लेने से मिलता है
[[File:finite-well-roots.gif|right|परिमाणित ऊर्जा स्तरों के लिए समीकरण की जड़ें]]
[[File:finite-well-roots.gif|right|परिमाणित ऊर्जा स्तरों के लिए समीकरण की जड़ें]]
<math display="block"> \alpha=k \tan(k L/2) .</math>
<math display="block"> \alpha=k \tan(k L/2) .</math>
इसी प्रकार एंटीसिमेट्रिक केस के लिए हमें मिलता है
इसी प्रकार एंटीसिमेट्रिक केस के लिए हमें मिलता है।<math display="block"> \alpha=-k \cot(k L/2) .</math>उस दोनों को याद करते है जो <math>\alpha</math> और <math>k</math> ऊर्जा पर निर्भर होते है. हमने पाया है कि ऊर्जा के अनैतिक मूल्य के लिए निरंतरता की शर्तों को संतुष्ट नहीं किया जा सकता है, जिससे कि यह अनंत संभावित कुएं की स्थितियों का परिणाम होता है। इस प्रकार, केवल कुछ ऊर्जा मान, जो इन दो समीकरणों में से या किसी का समाधान होता हैं, इसकी अनुमति देता है। इसलिए हम प्राप्त करते हैं कि सिस्टम का ऊर्जा स्तर से नीचे होता है और <math>V_0</math> से भिन्न होता हैं। इस प्रकार संबंधित आइजनफलन [[बाध्य अवस्था]]एँ हैं। (इसके विपरीत, उपरोक्त ऊर्जा स्तरों के लिए <math>V_0</math> निरंतर होता हैं।<ref>{{harvnb|Hall|2013}} Section 5.5</ref>)
<math display="block"> \alpha=-k \cot(k L/2) .</math>
 
उस दोनों को याद करें <math>\alpha</math> और <math>k</math> ऊर्जा पर निर्भर है. हमने पाया है कि ऊर्जा के मनमाने मूल्य के लिए निरंतरता की शर्तों को संतुष्ट नहीं किया जा सकता है; क्योंकि यह अनंत संभावित कुएं के मामले का परिणाम है। इस प्रकार, केवल कुछ ऊर्जा मान, जो इन दो समीकरणों में से एक या किसी एक का समाधान हैं, की अनुमति है। इसलिए हम पाते हैं कि सिस्टम का ऊर्जा स्तर नीचे है <math>V_0</math> अलग हैं; संबंधित eigenfunctions [[बाध्य अवस्था]]एँ हैं। (इसके विपरीत, उपरोक्त ऊर्जा स्तरों के लिए <math>V_0</math> निरंतर हैं.<ref>{{harvnb|Hall|2013}} Section 5.5</ref>)
ऊर्जा समीकरणों को विश्लेषणात्मक रूप से हल नहीं किया जा सकता है। सामान्यतः फिर भी, हम देख सकते है कि सममित स्थितियों में, सदैव कम से कम बंधी हुई स्थिति उपस्तिथ होती है, यदि कुआँ बहुत उथला होता है।<ref>{{harvnb|Hall|2013}} Proposition 5.3</ref>


ऊर्जा समीकरणों को विश्लेषणात्मक रूप से हल नहीं किया जा सकता है। फिर भी, हम देखेंगे कि सममित मामले में, हमेशा कम से कम एक बंधी हुई स्थिति मौजूद होती है, भले ही कुआँ बहुत उथला हो।<ref>{{harvnb|Hall|2013}} Proposition 5.3</ref>
ऊर्जा समीकरणों के आलेखीय या संख्यात्मक समाधानों को पुनः लिखने से सहायता मिलती है। यदि हम <math>u=\alpha L/2 </math> और <math>v=k L/2 </math> आयामहीन चर का परिचय देते हैं, और <math>\alpha</math> और <math>k</math> वह <math>u^2 = u_0^2-v^2</math>, जहाँ <math>u_0^2=m L^2 V_0/2 \hbar^2 </math> की परिभाषाओं पर ध्यान देते है, अतः मास्टर समीकरण पढ़ सकते है।
ऊर्जा समीकरणों के आलेखीय या संख्यात्मक समाधानों को थोड़ा पुनः लिखने से सहायता मिलती है। यदि हम आयामहीन चर का परिचय देते हैं <math>u=\alpha L/2 </math> और <math>v=k L/2 </math>, और की परिभाषाओं से ध्यान दें <math>\alpha</math> और <math>k</math> वह <math>u^2 = u_0^2-v^2</math>, कहाँ <math>u_0^2=m L^2 V_0/2 \hbar^2 </math>, मास्टर समीकरण पढ़ें
<math display="block">\sqrt{u_0^2-v^2} = \begin{cases}
<math display="block">\sqrt{u_0^2-v^2} = \begin{cases}
v \tan v, & \text{(symmetric case) } \\
v \tan v, & \text{(symmetric case) } \\
-v \cot v, & \text{(antisymmetric case) }
-v \cot v, & \text{(antisymmetric case) }
\end{cases}</math>
\end{cases}</math>
दाहिनी ओर के कथानक में, के लिए <math>u_0^2=20</math>, समाधान मौजूद हैं जहां नीला अर्धवृत्त बैंगनी या भूरे रंग के वक्रों को काटता है (<math>v \tan v</math> और <math>-v \cot v</math>). प्रत्येक बैंगनी या ग्रे वक्र एक संभावित समाधान का प्रतिनिधित्व करता है, <math>v_i</math> सीमा के अंदर <math display="inline">\frac{\pi}{2}(i-1) \leq v_i < \frac{\pi}{2}i</math>. समाधानों की कुल संख्या, <math>N</math>, (अर्थात, नीले वृत्त द्वारा प्रतिच्छेदित बैंगनी/ग्रे वक्रों की संख्या) इसलिए नीले वृत्त की त्रिज्या को विभाजित करके निर्धारित की जाती है, <math>u_0</math>, प्रत्येक समाधान की सीमा के अनुसार <math>\pi/2</math> और फर्श या छत के कार्यों का उपयोग करना:<ref>{{cite book|last=Williams|first=Floyd|title=क्वांटम यांत्रिकी में विषय| publisher=Springer Science+Business Media|year=2003|page=57|isbn=978-1-4612-6571-9|url=https://books.google.com/books?id=BovfBwAAQBAJ&q=number+of+bound+states+in+a+finite+potential+well&pg=PA57}}</ref>
दाहिनी ओर के कथानक में, <math>u_0^2=20</math> के लिए, समाधान उपस्तिथ होता हैं जहां नीला अर्धवृत्त बैंगनी या भूरे रंग के वक्रों को काटता है (<math>v \tan v</math> और <math>-v \cot v</math>)प्रत्येक बैंगनी या ग्रे वक्र संभावित समाधान का प्रतिनिधित्व करता है, <math>v_i</math> सीमा के अंदर <math display="inline">\frac{\pi}{2}(i-1) \leq v_i < \frac{\pi}{2}i</math>. समाधानों की कुल संख्या, <math>N</math>, (अर्थात्, नीले वृत्त द्वारा प्रतिच्छेदित बैंगनी/ग्रे वक्रों की संख्या) इसलिए नीले वृत्त की त्रिज्या <math>u_0</math> को विभाजित करके निर्धारित की जाती है, अतः प्रत्येक समाधान की सीमा के अनुसार <math>\pi/2</math> और फर्श या छत के कार्यों का उपयोग किया जाता है।<ref>{{cite book|last=Williams|first=Floyd|title=क्वांटम यांत्रिकी में विषय| publisher=Springer Science+Business Media|year=2003|page=57|isbn=978-1-4612-6571-9|url=https://books.google.com/books?id=BovfBwAAQBAJ&q=number+of+bound+states+in+a+finite+potential+well&pg=PA57}}</ref>
<math display="block">N = \left\lfloor\frac{2u_0}{\pi}\right\rfloor+1=\left\lceil\frac{2u_0}{\pi}\right\rceil</math>
<math display="block">N = \left\lfloor\frac{2u_0}{\pi}\right\rfloor+1=\left\lceil\frac{2u_0}{\pi}\right\rceil</math>
इस मामले में, वास्तव में तीन समाधान हैं <math>N = \lfloor 2\sqrt{20}/\pi\rfloor+1 = \lfloor 2.85 \rfloor+1 = 2+1 = 3</math>.
इस स्थितियों में, वास्तव में तीन समाधान होते हैं <math>N = \lfloor 2\sqrt{20}/\pi\rfloor+1 = \lfloor 2.85 \rfloor+1 = 2+1 = 3</math>.
[[File:finite-well-solutions.gif|right|परिमित वर्ग के समाधान अच्छी तरह से]]
[[File:finite-well-solutions.gif|right|परिमित वर्ग के समाधान अच्छी तरह से]]
<math>v_1 =1.28, v_2=2.54</math> और <math>v_3=3.73</math>, संगत ऊर्जाओं के साथ
<math>v_1 =1.28, v_2=2.54</math> और <math>v_3=3.73</math>, संगत ऊर्जाओं के साथ
  <math display="block">E_n={2\hbar^2 v_n^2\over m L^2} .</math>
  <math display="block">E_n={2\hbar^2 v_n^2\over m L^2} .</math>
यदि हम चाहें तो हम पीछे जाकर स्थिरांकों का मान ज्ञात कर सकते हैं <math>A, B, G, H</math> अब समीकरणों में (हमें सामान्यीकरण की स्थिति भी लागू करने की आवश्यकता है)। दाईं ओर हम इस मामले में ऊर्जा स्तर और तरंग कार्यों को दिखाते हैं (जहां)। <math display="inline">x_0\equiv\hbar/\sqrt{2m V_0}</math>):
यदि हम चाहें तब हम पीछे जाकर स्थिरांकों का मान ज्ञात कर सकते हैं <math>A, B, G, H</math> वर्तमान समीकरणों में (हमें सामान्यीकरण की स्थिति भी प्रयुक्त करने की आवश्यकता होती है)। इस प्रकार दाईं ओर हम इस स्थितियों में ऊर्जा स्तर और तरंग कार्यों को दिखाते हैं (जहां)। <math display="inline">x_0\equiv\hbar/\sqrt{2m V_0}</math>):


हम ध्यान दें कि यह कितना भी छोटा क्यों न हो <math>u_0</math> (चाहे कुआँ कितना भी उथला या संकरा क्यों न हो), वहाँ हमेशा कम से कम एक बंधी हुई अवस्था होती है।
ध्यान दीजिए कि <math>u_0</math> यह कितना भी छोटा क्यों न होता हो (चाहे कुआँ कितना भी उथला या संकरा क्यों न हो), वहाँ सदैव कम से कम बंधी हुई अवस्था होती है।


दो विशेष मामले ध्यान देने योग्य हैं। जैसे-जैसे क्षमता की ऊंचाई बड़ी होती जाती है, <math>V_0\to\infty</math>, अर्धवृत्त की त्रिज्या बड़ी हो जाती है और जड़ें मूल्यों के करीब और करीब आ जाती हैं <math>v_n=n\pi/2</math>, और हम अनंत वर्ग के मामले को अच्छी तरह से पुनर्प्राप्त करते हैं।
दो विशेष स्थिति ध्यान देने योग्य हैं। जैसे-जैसे क्षमता की ऊंचाई बड़ी होती जाती है, <math>V_0\to\infty</math>, अर्धवृत्त की त्रिज्या बड़ी हो जाती है और जड़ें मूल्यों के समीप और समीप आ जाती हैं <math>v_n=n\pi/2</math>, और हम अनंत वर्ग के स्थितियों को अच्छी प्रकार से पुनर्प्राप्त करते हैं।


दूसरा मामला एक बहुत ही संकीर्ण, गहरे कुएं का है - विशेष रूप से मामला <math>V_0\to\infty</math> और <math>L\to 0</math> साथ <math>V_0 L</math> हल किया गया। जैसा <math>u_0\propto \sqrt{V_0} L </math> यह शून्य की ओर प्रवृत्त होगा, और इसलिए केवल एक बंधी हुई अवस्था होगी। तब अनुमानित समाधान है <math>v^2 = u_0^2 - u_0^4</math>, और ऊर्जा प्रवृत्त होती है <math>E=-m L^2 V_0^2/2\hbar^2</math>. लेकिन यह केवल डेल्टा फ़ंक्शन क्षमता की बाध्य अवस्था की ऊर्जा है <math>V_0 L</math>, जैसा होना चाहिए।
दूसरी स्थिति बहुत ही संकीर्ण, गहरे कुएं की होती है - विशेष रूप से स्थिति <math>V_0\to\infty</math> और <math>L\to 0</math> के साथ <math>V_0 L</math> हल किया गया है। जैसा <math>u_0\propto \sqrt{V_0} L </math> यह शून्य की ओर प्रवृत्त होता है, और इसलिए केवल बंधी हुई अवस्था होती है। तब अनुमानित समाधान <math>v^2 = u_0^2 - u_0^4</math> होता है, और ऊर्जा प्रवृत्त <math>E=-m L^2 V_0^2/2\hbar^2</math> होती है। किन्तु यह केवल डेल्टा फलन क्षमता की बाध्य अवस्था की ऊर्जा <math>V_0 L</math> होती है, जैसा कि सामान्य रूप से होता है।


गुणन के माध्यम से क्षमता और ऊर्जा को सामान्य करके ऊर्जा स्तरों के लिए एक सरल ग्राफिकल समाधान प्राप्त किया जा सकता है <math>{8 m}{L^2} / h^2 </math>. सामान्यीकृत मात्राएँ हैं
सामान्यतः गुणन के माध्यम से क्षमता और ऊर्जा को सामान्य करके ऊर्जा स्तरों के लिए सरल ग्राफिकल समाधान प्राप्त किया जा सकता है <math>{8 m}{L^2} / h^2 </math>. सामान्यीकृत मात्राएँ होती हैं।
<math display="block"> \tilde{V}_0= V_0  \frac{8 m}{h^2} L^2 \qquad \tilde{E}= E  \frac{8 m}{h^2} L^2</math>
<math display="block"> \tilde{V}_0= V_0  \frac{8 m}{h^2} L^2 \qquad \tilde{E}= E  \frac{8 m}{h^2} L^2</math>
अनुमत जोड़ों के बीच सीधे संबंध देना <math> (V_0, E) </math> जैसा<ref>{{cite arXiv|last1=Chiani|first1=M. | title=वर्ग क्वांटम कुएं के ऊर्जा स्तर के लिए एक चार्ट|date=2016|eprint=1610.04468|class=physics.gen-ph}}</ref>
अनुमत जोड़ों के मध्य सीधे संबंध देना होता है <math> (V_0, E) </math> जैसा<ref>{{cite arXiv|last1=Chiani|first1=M. | title=वर्ग क्वांटम कुएं के ऊर्जा स्तर के लिए एक चार्ट|date=2016|eprint=1610.04468|class=physics.gen-ph}}</ref>
<math display="block"> \sqrt{\tilde{V}_0}={\sqrt{\tilde{E}}}\, \left|{\sec(\sqrt{\tilde{E}} \, {\pi}/{2})}\right|, \qquad \sqrt{\tilde{V}_0} = {\sqrt{\tilde{E}}}\, \left|{\csc(\sqrt{\tilde{E}} \, {\pi}/{2})}\right|</math>
<math display="block"> \sqrt{\tilde{V}_0}={\sqrt{\tilde{E}}}\, \left|{\sec(\sqrt{\tilde{E}} \, {\pi}/{2})}\right|, \qquad \sqrt{\tilde{V}_0} = {\sqrt{\tilde{E}}}\, \left|{\csc(\sqrt{\tilde{E}} \, {\pi}/{2})}\right|</math>
क्रमशः सम और विषम समता तरंग कार्यों के लिए। पिछले समीकरणों में केवल कार्यों के सकारात्मक व्युत्पन्न भागों पर विचार किया जाना है। चार्ट सीधे अनुमत जोड़ों को दे रहा है <math>(V_0, E) </math> चित्र में बताया गया है।
क्रमशः सम और विषम समता तरंग कार्यों के लिए पिछले समीकरणों में केवल कार्यों के धनात्मक व्युत्पन्न भागों पर विचार किया जाना है। इस प्रकार चार्ट सीधे अनुमत जोड़ों <math>(V_0, E) </math> को दे रहा है, जैसा कि चित्र में बताया गया है।


[[File:FigureV0E QuantumWell.png|thumb|upright=2.5]]
[[File:FigureV0E QuantumWell.png|thumb|upright=2.5]]
Line 122: Line 113:
===असंबद्ध अवस्थाएँ===
===असंबद्ध अवस्थाएँ===


यदि हम किसी ऊर्जा के लिए समय-स्वतंत्र श्रोडिंगर समीकरण को हल करते हैं <math>E > V_0</math>, समाधान कुएं के अंदर और बाहर दोनों जगह दोलनशील होंगे। इस प्रकार, समाधान कभी भी वर्ग पूर्णांक नहीं होता है; अर्थात्, यह हमेशा एक गैर-सामान्यीकरण योग्य स्थिति होती है। हालाँकि, इसका मतलब यह नहीं है कि क्वांटम कण के लिए इससे अधिक ऊर्जा होना असंभव है <math>V_0</math>, इसका मतलब केवल यह है कि सिस्टम के ऊपर निरंतर स्पेक्ट्रम है <math>V_0</math>. गैर-सामान्यीकरण योग्य ईजेनस्टेट वर्गाकार एकीकृत होने के काफी करीब हैं कि वे अभी भी एक असीमित ऑपरेटर के रूप में हैमिल्टनियन के स्पेक्ट्रम में योगदान करते हैं।<ref>{{harvnb|Hall|2013}} Section 5.5 and Exercise 4 in Chapter 3</ref>
यदि हम किसी ऊर्जा के लिए समय-स्वतंत्र श्रोडिंगर समीकरण को हल करते हैं <math>E > V_0</math>, समाधान कुएं के अंदर और बाहर दोनों स्थान दोलनशील होंगे। इस प्रकार, समाधान कभी भी वर्ग पूर्णांक नहीं होता है; अर्थात्, यह सदैव गैर-सामान्यीकरण योग्य स्थिति होती है। चूँकि, इसका कारण यह नहीं होता है कि क्वांटम कण के लिए इससे अधिक ऊर्जा <math>V_0</math> होना असंभव है, इसका कारण केवल यह होता है कि सिस्टम के ऊपर निरंतर वर्णक्रम <math>V_0</math> है। इस प्रकार गैर-सामान्यीकरण योग्य ईजेनस्टेट वर्गाकार एकीकृत होने के अधिक समीप होता हैं कि वह अभी भी असीमित ऑपरेटर के रूप में हैमिल्टनियन के वर्णक्रम में योगदान करते हैं।<ref>{{harvnb|Hall|2013}} Section 5.5 and Exercise 4 in Chapter 3</ref>
 
 
==असममित कुआँ==
==असममित कुआँ==
क्षमता द्वारा अच्छी तरह से दी गई एक-आयामी असममित क्षमता पर विचार करें<ref>Landau, L. D., & Lifshitz, E. M. (2013). Quantum mechanics: non-relativistic theory (Vol. 3). Elsevier.</ref>
सामान्यतः क्षमता द्वारा अच्छी प्रकार से दी गई एक-आयामी असममित क्षमता पर विचार करते है।<ref>Landau, L. D., & Lifshitz, E. M. (2013). Quantum mechanics: non-relativistic theory (Vol. 3). Elsevier.</ref>


<math display="block">V(x) = \begin{cases}
<math display="block">V(x) = \begin{cases}
Line 133: Line 122:
V_2 & \text{if }a<x<\infty\text{  (the region outside the box)}
V_2 & \text{if }a<x<\infty\text{  (the region outside the box)}
\end{cases}</math>
\end{cases}</math>
साथ <math>V_2 > V_1</math>. तरंग फ़ंक्शन के लिए संगत समाधान <math>E<V_1</math> होना पाया जाता है
साथ <math>V_2 > V_1</math> तरंग फलन के लिए संगत समाधान <math>E<V_1</math> होना पाया जाता है।


<math display="block">\psi(x) = \begin{cases}
<math display="block">\psi(x) = \begin{cases}
Line 142: Line 131:
और
और
<math display="block">\sin\delta = \frac{k\hbar}{\sqrt{2mV_1}}.</math>
<math display="block">\sin\delta = \frac{k\hbar}{\sqrt{2mV_1}}.</math>
ऊर्जा का स्तर <math>E=k^2\hbar^2/(2m)</math> एक बार निर्धारित किया जाता है <math>k</math> निम्नलिखित पारलौकिक समीकरण के मूल के रूप में हल किया गया है
ऊर्जा का स्तर <math>E=k^2\hbar^2/(2m)</math> प्रत्येक बार निर्धारित किया जाता है, अतः <math>k</math> निम्नलिखित पारलौकिक समीकरण के मूल के रूप में हल किया गया है।


<math display="block">ka = n\pi - \sin^{-1}\left(\frac{k\hbar}{\sqrt{2mV_1}}\right) - \sin^{-1}\left(\frac{k\hbar}{\sqrt{2mV_2}}\right)</math>
<math display="block">ka = n\pi - \sin^{-1}\left(\frac{k\hbar}{\sqrt{2mV_1}}\right) - \sin^{-1}\left(\frac{k\hbar}{\sqrt{2mV_2}}\right)</math>
कहाँ <math>n=1,2,3,\dots</math> उपरोक्त समीकरण के मूल के अस्तित्व की हमेशा गारंटी नहीं होती है, उदाहरण के लिए, कोई हमेशा इसका मान पा सकता है <math>a</math> इतना छोटा, कि दिए गए मानों के लिए <math>V_1</math> और <math>V_2</math>, कोई पृथक ऊर्जा स्तर मौजूद नहीं है। सममित कुएं के परिणाम उपरोक्त समीकरण से सेटिंग द्वारा प्राप्त किये जाते हैं <math>V_1 = V_2 = V_o</math>.
जहाँ <math>n=1,2,3,\dots</math> उपरोक्त समीकरण के मूल "के" अस्तित्व की सदैव गारंटी नहीं होती है, उदाहरण के लिए, कोई सदैव इसका मान प्राप्त कर सकता है, अतः <math>a</math> इतना छोटा होता है कि दिए गए मानों के लिए <math>V_1</math> और <math>V_2</math>, कोई पृथक ऊर्जा स्तर उपस्तिथ नहीं होती है। इस प्रकार सममित कुएं के परिणाम उपरोक्त समीकरण से समुच्चय द्वारा <math>V_1 = V_2 = V_o</math> प्राप्त किये जाते हैं।


==गोलाकार गुहा==
==गोलाकार गुहा==


उपरोक्त परिणामों का उपयोग यह दिखाने के लिए किया जा सकता है कि, एक-आयामी मामले में, गोलाकार गुहा में दो बाध्य अवस्थाएँ होती हैं, क्योंकि गोलाकार निर्देशांक किसी भी दिशा में त्रिज्या के बराबर बनाते हैं।
उपरोक्त परिणामों का उपयोग यह दिखाने के लिए किया जा सकता है कि, एक-आयामी स्थितियों में, गोलाकार गुहा में दो बाध्य अवस्थाएँ होती हैं, जिससे कि गोलाकार निर्देशांक किसी भी दिशा में त्रिज्या के सामान्तर बनाते हैं।


गोलाकार रूप से सममित क्षमता की जमीनी स्थिति (n = 1) में हमेशा शून्य कक्षीय कोणीय गति (ℓ = n−1) होगी, और कम तरंग फ़ंक्शन होगा
गोलाकार रूप से सममित क्षमता की जमीनी स्थिति (''N'' = 1) में सदैव शून्य कक्षीय कोणीय गति (ℓ = ''N''−1) होती है, और निम्न तरंग फलन होता है।


<math>{\displaystyle U(r)\equiv r\psi (r)}</math>
<math>{\displaystyle U(r)\equiv r\psi (r)}</math>
समीकरण को संतुष्ट करता है
 
समीकरण को संतुष्ट करता है।


<math>  {\displaystyle -{\frac {\hbar ^{2}}{2m}}{\frac {d^{2}U}{dr^{2}}}+V(r)U(r)=EU(r)}</math>
<math>  {\displaystyle -{\frac {\hbar ^{2}}{2m}}{\frac {d^{2}U}{dr^{2}}}+V(r)U(r)=EU(r)}</math>
कहाँ <math>\psi (r)</math> तरंग फ़ंक्शन का रेडियल भाग है। ध्यान दें कि (n = 1) के लिए कोणीय भाग स्थिर है (ℓ = 0)।


सीमा स्थितियों को छोड़कर, यह एक-आयामी समीकरण के समान है। पहले जैसा,
जहाँ <math>\psi (r)</math> तरंग फलन का रेडियल भाग होता है। ध्यान दीजिए कि (''N'' = 1) के लिए कोणीय भाग स्थिर होता है (ℓ = 0)।
 
सीमा स्थितियों को छोड़कर, यह एक-आयामी समीकरण के समान होता है। पहले जैसा,


<math>{\displaystyle U (r)={\begin{cases}c_{1}\sin({k_{1}r}),&{\text{for }} r<a,{\text{where }} k_{1}={\sqrt {2m /\hbar ^{2}(V_{1}-E)}} \\c\sin(kr+\delta ),&{\text{for }}a<r< b,{\text{where }}k={\sqrt {2mE/\hbar ^{2}}}\\c_{2}e^{-k_{2}r},&{\text{for }}r>b,{\text{where }}k_{2}={\sqrt {2m /\hbar ^{2}(V_{2}-E)}}\end{cases}}}
<math>{\displaystyle U (r)={\begin{cases}c_{1}\sin({k_{1}r}),&{\text{for }} r<a,{\text{where }} k_{1}={\sqrt {2m /\hbar ^{2}(V_{1}-E)}} \\c\sin(kr+\delta ),&{\text{for }}a<r< b,{\text{where }}k={\sqrt {2mE/\hbar ^{2}}}\\c_{2}e^{-k_{2}r},&{\text{for }}r>b,{\text{where }}k_{2}={\sqrt {2m /\hbar ^{2}(V_{2}-E)}}\end{cases}}}
</math>
</math>
के लिए ऊर्जा स्तर <math>a<r< b
 
इसके लिए ऊर्जा स्तर <math>a<r< b
</math>
</math>


<math>{\displaystyle E=k^{2}\hbar ^{2}/(2m)}
<math>{\displaystyle E=k^{2}\hbar ^{2}/(2m)}
</math>
</math>
एक बार निर्धारित किया जाता है
 
यह प्रत्येक बार निर्धारित किया जाता है।


<math>  {\displaystyle k}</math>
<math>  {\displaystyle k}</math>
निम्नलिखित पारलौकिक समीकरण के मूल के रूप में हल किया गया है
 
निम्नलिखित पारलौकिक समीकरण के मूल के रूप में हल किया गया है।


<math>  {\displaystyle k(b-a)=n\pi}</math>
<math>  {\displaystyle k(b-a)=n\pi}</math>
कहाँ
 
जहाँ


<math>  {\displaystyle n=1,2,3,\dots }</math>
<math>  {\displaystyle n=1,2,3,\dots }</math>
उपरोक्त समीकरण के मूल के अस्तित्व की हमेशा गारंटी होती है।


परिणाम हमेशा गोलाकार समरूपता के साथ होते हैं।
उपरोक्त समीकरण के मूल के अस्तित्व की सदैव गारंटी होती है।


यह उस स्थिति को पूरा करता है जहां तरंग को गोले के अंदर कोई क्षमता नहीं मिलती है: <math>  {\displaystyle U(a) = U(0)=0}</math>
परिणाम सदैव गोलाकार समरूपता के साथ होते हैं।


 
यह उस स्थिति को पूर्ण करता है जहां तरंग को गोले के अंदर कोई क्षमता <math>  {\displaystyle U(a) = U(0)=0}</math> नहीं मिलती है।
==यह भी देखें==
=='''यह भी देखें'''==
*संभावित कुआँ
*संभावित कुआँ
*डेल्टा कार्य क्षमता
*डेल्टा कार्य क्षमता
*अनंत क्षमता वाला कुँआ
*अनंत क्षमता वाला स्रोत
*अर्धवृत्त क्षमता अच्छी तरह से
*अर्धवृत्त क्षमता अच्छी प्रकार से
*क्वांटम टनलिंग
*क्वांटम टनलिंग
*[[आयताकार संभावित अवरोध]]
*[[आयताकार संभावित अवरोध]]


==संदर्भ==
=='''संदर्भ'''==
{{Reflist}}
{{Reflist}}
 
=='''अग्रिम पठन'''==
 
==अग्रिम पठन==
*{{cite book
*{{cite book
  | author=Griffiths, David J. |authorlink=David J. Griffiths
  | author=ग्रिफ़िथ्स, डेविड जे. |authorlink=डेविड जे. ग्रिफ़िथ्स
  | year=2005
  | year=2005
  | title=Introduction to Quantum Mechanics
  | title=क्वांटम यांत्रिकी का परिचय
  | edition = 2nd
  | edition = 2nd
  | publisher=[[Prentice-Hall]]
  | publisher=[[शागिर्द कक्ष]]
  | isbn=0-13-111892-7
  | isbn=0-13-111892-7
}}
}}
* {{citation|first=Brian C.|last=Hall|title=Quantum Theory for Mathematicians|series=Graduate Texts in Mathematics|volume=267 |publisher=Springer|year=2013}}.
* {{citation|first=ब्रायन सी.|last=बड़ा कमरा|title=गणितज्ञों के लिए क्वांटम सिद्धांत|series=गणित में स्नातक पाठ|volume=267 |publisher=कोंपल|year=2013}}.
[[Category: क्वांटम यांत्रिक क्षमताएँ]] [[Category: क्वांटम मॉडल]] [[Category: बिल्कुल हल करने योग्य मॉडल]]
 
 


[[Category: Machine Translated Page]]
[[Category:Created On 26/07/2023]]
[[Category:Created On 26/07/2023]]
[[Category:Machine Translated Page]]
[[Category:Pages with script errors]]
[[Category:Templates Vigyan Ready]]
[[Category:क्वांटम मॉडल]]
[[Category:क्वांटम यांत्रिक क्षमताएँ]]
[[Category:बिल्कुल हल करने योग्य मॉडल]]

Latest revision as of 11:27, 14 August 2023

परिमित संभावित स्रोत (परिमित वर्ग स्रोत के रूप में भी जाना जाता है) क्वांटम यांत्रिकी की अवधारणा होती है। यह अनंत क्षमता वाले कुएं का विस्तार होता है, जिसमें कण "बॉक्स" तक ही सीमित होता है, किन्तु जिसकी संभावित ऊर्जा "दीवारें" सीमित होती हैं। इस प्रकार अनंत क्षमता वाले कुएं के विपरीत, कण के बॉक्स के बाहर पाए जाने से जुड़ी संभावना होती है। चूँकि क्वांटम यांत्रिक व्याख्या मौलिक व्याख्या के विपरीत होती है, जहां यदि कण की कुल ऊर्जा दीवारों की संभावित ऊर्जा बाधा से कम है तब इसे बॉक्स के बाहर नहीं पाया जा सकता है। इस प्रकार क्वांटम व्याख्या में, कण की ऊर्जा दीवारों की संभावित ऊर्जा बाधा (सीएफ क्वांटम टनलिंग) से कम होने पर भी कण के बॉक्स के बाहर होने की गैर-शून्य संभावना होती है।

एक-आयामी बॉक्स में कण

एक्स-अक्ष पर 1-आयामी स्थितियों के लिए, समय-स्वतंत्र श्रोडिंगर समीकरण को इस प्रकार लिखा जा सकता है।

 

 

 

 

(1)

जहाँ

  • घटा हुआ प्लैंक स्थिरांक होता है,
  • प्लैंक स्थिरांक होता है,
  • कण का द्रव्यमान होता है,
  • वह (समष्टि मूल्यवान) तरंग क्रिया होती है जिसे हम खोजना चाहते हैं,
  • प्रत्येक बिंदु एक्स पर संभावित ऊर्जा का वर्णन करने वाला फलन होता है, और
  • ऊर्जा होती है, वास्तविक संख्या, जिसे कभी-कभी आइजेनएनर्जी भी कहा जाता है।

लंबाई एल के 1-आयामी बॉक्स में कण की स्थितियों में, क्षमता होती है। इस प्रकार बॉक्स के बाहर, और मध्य में एक्स के लिए शून्य और . तरंग फलन को एक्स की विभिन्न श्रेणियों पर भिन्न-भिन्न तरंग फलन से बना माना जाता है, यह इस पर निर्भर करता है कि एक्स बॉक्स के अंदर या बाहर होता है। इसलिए, तरंग फलन को इस प्रकार परिभाषित किया गया है।

बॉक्स के अंदर के क्षेत्र के लिए, वी(एक्स) = 0 और समीकरण 1 कम हो जाता है
दे
समीकरण बन जाता है
यह सामान्य समाधान के साथ अच्छी प्रकार से अध्ययन किया गया अंतर समीकरण और आइजेनवेक्टर समस्या होती है
इस प्रकार,


यहां, ए और बी कोई भी सम्मिश्र संख्या हो सकती हैं, और "के" कोई भी वास्तविक संख्या हो सकती है।

बॉक्स के बाहर

बॉक्स के बाहर के क्षेत्र के लिए और समीकरण 1 बन जाता है, चूँकि क्षमता स्थिर होती है।

सामान्यतः समाधान के दो संभावित समूह होते हैं, यह इस पर निर्भर करता है कि ई इससे कम होता है या नहीं होता है (कण विभव में बंधा हुआ है) अथवा ई से अधिक (कण स्वतंत्र) होता है।

मुक्त कण के लिए, , और देना

का उत्पादन
आंतरिक अच्छी प्रकार की स्थिति के समान समाधान फॉर्म के साथ:

यह विश्लेषण बाध्य स्थिति पर ध्यान केंद्रित करता है, जहां देता है।
का उत्पादन
जहां सामान्य समाधान घातीय होता है।
इसी प्रकार, बॉक्स के बाहर दूसरे क्षेत्र के लिए:

वर्तमान उपस्तिथ समस्या का विशिष्ट समाधान खोजने के लिए, हमें उपयुक्त सीमा शर्तों को निर्दिष्ट करना होता है और ए, बी, एफ, जी, एच और आई के लिए मान खोजना होता है, जो उन शर्तों को पूर्ण करते हैं।

बाउंड अवस्था के लिए तरंग फलन खोजना

श्रोडिंगर समीकरण के समाधान निरंतर और निरंतर भिन्न होते है।[1] यह आवश्यकताएं पहले से प्राप्त अंतर समीकरणों पर सीमा की स्थिति होती हैं, अर्थात् कुएं के अंदर और बाहर के समाधानों के मध्य मिलान की स्थिति होती है।

इस स्थितियों में, परिमित संभावित कुआं सममित होता है, इसलिए आवश्यक गणनाओं को कम करने के लिए समरूपता का उपयोग किया जा सकता है।

पिछले अनुभागों का सारांश:

जहां हमें , , और प्राप्त होता है।

हम इसे ऐसे देखते हैं. जंहा जाता है तक, जंहा पद अनंत तक जाता है. इसी प्रकार, जैसे जाता है तक, उसी प्रकार पद अनंत तक जाता है। सामान्यतः तरंग फलन को वर्गाकार समाकलनीय बनाने के लिए, हमें समुच्चय करना होता है, और हमारे पास होता है।

और


अगला, हम जानते हैं कि समग्र फलन निरंतर और भिन्न होता है। दूसरे शब्दों में, फलन और उनके व्युत्पन्न के मान विभाजन बिंदुओं पर मेल खाते है।

इन समीकरणों के दो प्रकार के समाधान होते हैं, अतः सममित, जिसके लिए और , और एंटीसिमेट्रिक, जिसके लिए और . सममित स्थितियों के लिए हमें मिलता है।

तब अनुपात लेने से मिलता है

परिमाणित ऊर्जा स्तरों के लिए समीकरण की जड़ें

इसी प्रकार एंटीसिमेट्रिक केस के लिए हमें मिलता है।
उस दोनों को याद करते है जो और ऊर्जा पर निर्भर होते है. हमने पाया है कि ऊर्जा के अनैतिक मूल्य के लिए निरंतरता की शर्तों को संतुष्ट नहीं किया जा सकता है, जिससे कि यह अनंत संभावित कुएं की स्थितियों का परिणाम होता है। इस प्रकार, केवल कुछ ऊर्जा मान, जो इन दो समीकरणों में से या किसी का समाधान होता हैं, इसकी अनुमति देता है। इसलिए हम प्राप्त करते हैं कि सिस्टम का ऊर्जा स्तर से नीचे होता है और से भिन्न होता हैं। इस प्रकार संबंधित आइजनफलन बाध्य अवस्थाएँ हैं। (इसके विपरीत, उपरोक्त ऊर्जा स्तरों के लिए निरंतर होता हैं।[2])

ऊर्जा समीकरणों को विश्लेषणात्मक रूप से हल नहीं किया जा सकता है। सामान्यतः फिर भी, हम देख सकते है कि सममित स्थितियों में, सदैव कम से कम बंधी हुई स्थिति उपस्तिथ होती है, यदि कुआँ बहुत उथला होता है।[3]

ऊर्जा समीकरणों के आलेखीय या संख्यात्मक समाधानों को पुनः लिखने से सहायता मिलती है। यदि हम और आयामहीन चर का परिचय देते हैं, और और वह , जहाँ की परिभाषाओं पर ध्यान देते है, अतः मास्टर समीकरण पढ़ सकते है।

दाहिनी ओर के कथानक में, के लिए, समाधान उपस्तिथ होता हैं जहां नीला अर्धवृत्त बैंगनी या भूरे रंग के वक्रों को काटता है ( और )। प्रत्येक बैंगनी या ग्रे वक्र संभावित समाधान का प्रतिनिधित्व करता है, सीमा के अंदर . समाधानों की कुल संख्या, , (अर्थात्, नीले वृत्त द्वारा प्रतिच्छेदित बैंगनी/ग्रे वक्रों की संख्या) इसलिए नीले वृत्त की त्रिज्या को विभाजित करके निर्धारित की जाती है, अतः प्रत्येक समाधान की सीमा के अनुसार और फर्श या छत के कार्यों का उपयोग किया जाता है।[4]
इस स्थितियों में, वास्तव में तीन समाधान होते हैं .

परिमित वर्ग के समाधान अच्छी तरह से

और , संगत ऊर्जाओं के साथ

यदि हम चाहें तब हम पीछे जाकर स्थिरांकों का मान ज्ञात कर सकते हैं वर्तमान समीकरणों में (हमें सामान्यीकरण की स्थिति भी प्रयुक्त करने की आवश्यकता होती है)। इस प्रकार दाईं ओर हम इस स्थितियों में ऊर्जा स्तर और तरंग कार्यों को दिखाते हैं (जहां)। ):

ध्यान दीजिए कि यह कितना भी छोटा क्यों न होता हो (चाहे कुआँ कितना भी उथला या संकरा क्यों न हो), वहाँ सदैव कम से कम बंधी हुई अवस्था होती है।

दो विशेष स्थिति ध्यान देने योग्य हैं। जैसे-जैसे क्षमता की ऊंचाई बड़ी होती जाती है, , अर्धवृत्त की त्रिज्या बड़ी हो जाती है और जड़ें मूल्यों के समीप और समीप आ जाती हैं , और हम अनंत वर्ग के स्थितियों को अच्छी प्रकार से पुनर्प्राप्त करते हैं।

दूसरी स्थिति बहुत ही संकीर्ण, गहरे कुएं की होती है - विशेष रूप से स्थिति और के साथ हल किया गया है। जैसा यह शून्य की ओर प्रवृत्त होता है, और इसलिए केवल बंधी हुई अवस्था होती है। तब अनुमानित समाधान होता है, और ऊर्जा प्रवृत्त होती है। किन्तु यह केवल डेल्टा फलन क्षमता की बाध्य अवस्था की ऊर्जा होती है, जैसा कि सामान्य रूप से होता है।

सामान्यतः गुणन के माध्यम से क्षमता और ऊर्जा को सामान्य करके ऊर्जा स्तरों के लिए सरल ग्राफिकल समाधान प्राप्त किया जा सकता है . सामान्यीकृत मात्राएँ होती हैं।

अनुमत जोड़ों के मध्य सीधे संबंध देना होता है जैसा[5]
क्रमशः सम और विषम समता तरंग कार्यों के लिए पिछले समीकरणों में केवल कार्यों के धनात्मक व्युत्पन्न भागों पर विचार किया जाना है। इस प्रकार चार्ट सीधे अनुमत जोड़ों को दे रहा है, जैसा कि चित्र में बताया गया है।

FigureV0E QuantumWell.png

असंबद्ध अवस्थाएँ

यदि हम किसी ऊर्जा के लिए समय-स्वतंत्र श्रोडिंगर समीकरण को हल करते हैं , समाधान कुएं के अंदर और बाहर दोनों स्थान दोलनशील होंगे। इस प्रकार, समाधान कभी भी वर्ग पूर्णांक नहीं होता है; अर्थात्, यह सदैव गैर-सामान्यीकरण योग्य स्थिति होती है। चूँकि, इसका कारण यह नहीं होता है कि क्वांटम कण के लिए इससे अधिक ऊर्जा होना असंभव है, इसका कारण केवल यह होता है कि सिस्टम के ऊपर निरंतर वर्णक्रम है। इस प्रकार गैर-सामान्यीकरण योग्य ईजेनस्टेट वर्गाकार एकीकृत होने के अधिक समीप होता हैं कि वह अभी भी असीमित ऑपरेटर के रूप में हैमिल्टनियन के वर्णक्रम में योगदान करते हैं।[6]

असममित कुआँ

सामान्यतः क्षमता द्वारा अच्छी प्रकार से दी गई एक-आयामी असममित क्षमता पर विचार करते है।[7]

साथ तरंग फलन के लिए संगत समाधान होना पाया जाता है।

और
ऊर्जा का स्तर प्रत्येक बार निर्धारित किया जाता है, अतः निम्नलिखित पारलौकिक समीकरण के मूल के रूप में हल किया गया है।

जहाँ उपरोक्त समीकरण के मूल "के" अस्तित्व की सदैव गारंटी नहीं होती है, उदाहरण के लिए, कोई सदैव इसका मान प्राप्त कर सकता है, अतः इतना छोटा होता है कि दिए गए मानों के लिए और , कोई पृथक ऊर्जा स्तर उपस्तिथ नहीं होती है। इस प्रकार सममित कुएं के परिणाम उपरोक्त समीकरण से समुच्चय द्वारा प्राप्त किये जाते हैं।

गोलाकार गुहा

उपरोक्त परिणामों का उपयोग यह दिखाने के लिए किया जा सकता है कि, एक-आयामी स्थितियों में, गोलाकार गुहा में दो बाध्य अवस्थाएँ होती हैं, जिससे कि गोलाकार निर्देशांक किसी भी दिशा में त्रिज्या के सामान्तर बनाते हैं।

गोलाकार रूप से सममित क्षमता की जमीनी स्थिति (N = 1) में सदैव शून्य कक्षीय कोणीय गति (ℓ = N−1) होती है, और निम्न तरंग फलन होता है।

समीकरण को संतुष्ट करता है।

जहाँ तरंग फलन का रेडियल भाग होता है। ध्यान दीजिए कि (N = 1) के लिए कोणीय भाग स्थिर होता है (ℓ = 0)।

सीमा स्थितियों को छोड़कर, यह एक-आयामी समीकरण के समान होता है। पहले जैसा,

इसके लिए ऊर्जा स्तर

यह प्रत्येक बार निर्धारित किया जाता है।

निम्नलिखित पारलौकिक समीकरण के मूल के रूप में हल किया गया है।

जहाँ

उपरोक्त समीकरण के मूल के अस्तित्व की सदैव गारंटी होती है।

परिणाम सदैव गोलाकार समरूपता के साथ होते हैं।

यह उस स्थिति को पूर्ण करता है जहां तरंग को गोले के अंदर कोई क्षमता नहीं मिलती है।

यह भी देखें

  • संभावित कुआँ
  • डेल्टा कार्य क्षमता
  • अनंत क्षमता वाला स्रोत
  • अर्धवृत्त क्षमता अच्छी प्रकार से
  • क्वांटम टनलिंग
  • आयताकार संभावित अवरोध

संदर्भ

  1. Hall 2013 Proposition 5.1
  2. Hall 2013 Section 5.5
  3. Hall 2013 Proposition 5.3
  4. Williams, Floyd (2003). क्वांटम यांत्रिकी में विषय. Springer Science+Business Media. p. 57. ISBN 978-1-4612-6571-9.
  5. Chiani, M. (2016). "वर्ग क्वांटम कुएं के ऊर्जा स्तर के लिए एक चार्ट". arXiv:1610.04468 [physics.gen-ph].
  6. Hall 2013 Section 5.5 and Exercise 4 in Chapter 3
  7. Landau, L. D., & Lifshitz, E. M. (2013). Quantum mechanics: non-relativistic theory (Vol. 3). Elsevier.

अग्रिम पठन